<<

QUESTIONS 1 3 e... )... s... broad QRS complexes broad persistent ST-segment persistent Æ Æ irregularly irregular P-waves ST-segment depression in leads depression ST-segment Æ Æ downward-sloping ST-segment downward-sloping Æ he syndrome is arterial occlusive disease below is arterialhe syndrome occlusive c ncerning atrialncerning fibrillation (AF ncerning aorticncerning stenosi peripheral diseas arterial occlusive

ri ntricular aneurysm ithin it is described the Fick principle by est ’commonly in the feet and occurs xpiration Co The following cardiac events are matched correctly matched are events The following cardiac In Co Dressler’s syndrome Dressler’s disociation electromechanical left ventricular aneurysm II,III and aVF pericarditis leads all anterior in chest depression atrial fibrillation dehydration caused by it is commonly inferior infarct heartfirst degree block ve elevation leads in chest the relationship between radius between ofthe relationship and flow w ‘cramping’inclaudication means Latin Le best with heard the are murmurs all patient on their left side with held in end- breath e ofit causes hypertrophy only the left ventricle in the radial‘Waterhammer’character it causes a systolic’murmur‘ejection it causes an cause ofit is a common atrial fibrillation the popliteal trifurcation the popliteal ‘r metatarsal heads usually painless arterial are ulcers the corresponding ECG changesthe corresponding . ..

(c) (d) (e) [8] [5] [6] [7] (b) (c) (a) to (a) (d) (e) (a) (b) (c) (a) (b) (c) (d) (e) (d) (e) Questions I... ncerning hypertensionncerning . . . ncerning cardiac failurecardiac ncerning . . . ute myocardial ute . . . aper 1 aper pothyroidism does not cause hypertension pothyroidism M hest radiograph aised levels ofaised levels atrial natriuretic peptide (ANP) are The following are recognized complications of complications recognized The following are Ac Co Co e Sheehan’s syndrome Sheehan’s aorticacute regurgitation causes not relieved by glyceryl by trinitrate pain not relieved causes chest (GTN) painless in elderly diabetic patients can be entirely administrationshould prompt of intramuscular pain relieve morphine to can be diagnosed on the history alone on ECG in the form of‘Q’wavesfirst shows approximately 50% ofapproximately all cases of hypertension are dysfunction renal caused by a raised usually have syndrome patients with Conn’s serum potassium level hy reflex’is‘Cushing blood in systolic an increase the secondary Cushing’s that occurs to pressure syndrome malignant hypertension can be diagnosed on alone fundoscopy all diuretics used in treatment ofall diuretics failure cardiac cause hypokalaemia potentially valvular disease is the most common cause in the UK common disease is the most valvular r thought be beneficial in to failure heart finding on a third sound is a common ausculation the heart looks small on plain postero-anterior c P ut (a) (b) ac (a) (b) (c) (d) (e) (a) (b) (c) (d) (e) (e) (a) (b) (c) (d) [4] [3] [2] MCQs [1] CUM1 2/15/05 4:33 PM Page 3 PM Page 4:33 2/15/05 CUM1 CUM1 2/15/05 4:33 PM Page 4

4 Paper 1 MCQs

(b) when caused by high levels of circulating thyroid (e) it causes ‘bat’s wing’pulmonary oedema,upper lobe hormone,it is always accompanied by thyroid eye diversion and Kerley-B lines on chest radiograph signs (c) no P waves are present on the ECG [13] Concerning asthma... (d) the QRS complex is broad (a) airway limitation is uniformly reversible in asthma

QUESTIONS 1 (e) class Ic antidysrhythmics (e.g.flecainide) are safe to (b) typical asthma attacks occur during exercise give to most patients with atrial tachyarrhythmias (c) ACE inhibitors should not be given to asthmatic subjects [9] Concerning the respiratory organs of the body . . . (d) symptoms of asthma are usually worse at night

(a) the right main bronchus lies more vertically than (e) b2 adrenoceptor antagonists are commonly used in the left treatment of asthma (b) the is an organ with a single blood supply (i.e. the pulmonary artery) [14] Tuberculosis... (c) the alveolar lining of the comprises mainly (a) is caused by the Gram-negative bacillus type I pneumocytes M. tuberculosis (d) only the right lung has an oblique fissure (b) is a recognized cause of apical lung fibrosis (e) in healthy subjects,the strongest stimulator of (c) is classically associated with the presence of ventilation is an increase in the arterial partial non-caseating granulomas

pressure of CO2 (PaCO2) (d) is diagnosed using one test with two different names: Heaf and Mantoux [10] On examination of the chest of a patient with a (e) is a recognized cause of erythema nodosum unilateral tension pneumothorax . . . (a) chest wall movements are decreased on both sides [15] Concerning palpation of the abdomen . . . (b) the mediastinum is pulled towards the affected (a) in healthy,slim individuals,the is normally side palpable in the right upper quadrant in deep (c) the percussion note may be hyperresonant inspiration (d) breath sounds may be decreased or absent on the (b) the normal liver extends superiorly to the 3rd affected side intercostal space (e) tactile vocal resonance will be increased on the (c) hepatic enlargement usually proceeds toward the affected side left iliac fossa (d) in healthy,slim individuals,the is normally [11] Concerning chronic obstructive pulmonary palpable on deep inspiration in the left disease (COPD)... hypochondrium (a) the airway obstruction is entirely irreversible (e) splenic enlargement occurs obliquely,toward the (b) chronic bronchitis is believed to be due mainly to right iliac fossa mucous gland hypertrophy (c) patients commonly demonstrate asterixis [16] The following are recognized gastrointestinal (d) ‘cor pulmonale’is defined as lung disease secondary causes of finger clubbing... to ventricular hypertrophy (a) fibrosis (e) it mainly leads to type II respiratory failure (b) coeliac disease (c) Plummer–Vinson syndrome [12] Concerning respiratory failure . . . (d) gastrointestinal lymphoma

(a) PaCO2 is,by definition,elevated in all types of (e) cystic fibrosis respiratory failure (b) the commonest cause of type II respiratory failure is [17] The following are recognized medical causes of pneumonia ... (c) is a sign commonly associated (a) diabetic ketoacidosis with respiratory failure (b) myocardial infarction (d) it always presents with dyspnoea (c) Addison’s disease CUM1 2/15/05 4:33 PM Page 6

6 Paper 1 EMQs

EMQs

Question 1 (a) Mitral stenosis (b) Mitral regurgitation Match the following examination findings to the most QUESTIONS 1 (c) Aortic regurgitation accurate diagnosis: (d) Aortic sclerosis with atrial fibrillation (e) Aortic regurgitation with left ventricular failure (1) JVP Normal (f) Mitral stenosis with atrial fibrillation Apex ‘Tapping’in character,not (g) Mitral regurgitation and stenosis displaced laterally (h) Grade 3 mid-diastolic murmur (i) Aortic sclerosis at apex,no pre-systolic (j) Mitral regurgitation with diastolic flow murmur accentuation (k) None of these Chest Clear BP 155/82 mmHg Question 2 (2) JVP Elevated Match the following case histories to the best diagnosis Apex Displaced laterally,and ‘thrust- given in the list below. ing’in character Auscultation Pan-systolic murmur at apex (1) A 47-year-old woman presents with a history of re- Chest Bi-basal end expiratory current over 5 years. The pain is described as crepitations unilateral, behind the eye and severe. She is well between BP 143/65 mmHg episodes. There is no vomiting although she feels nau- seated and has abdominal discomfort. The onset is typi- (3) JVP Normal cally sudden. She occasionally has ‘hazy’ disturbance of Apex ‘Tapping’in character,not her vision. There are no abnormal features on systemic displaced review. In the past she has had hypertension, but is cur- Auscultation Grade 3 mid-diastolic murmur rently not taking treatment.She has three children.There at apex,pre-systolic are no regular medications. On examination she has a accentuation BMI of 29 and a BP recording of 170/95 mmHg. Other- Chest Clear wise a full neurological examination was normal,includ- BP 162/73 mmHg ing fundoscopy.

(4) JVP 8 cm (2) A 15-year-old girl presents with a 5-week history of Apex Displaced,thrusting in character persistent headaches. These are described as ‘all over the Auscultation Short early diastolic murmur head’.There is associated episodic blurring of vision and Chest Few bi-basal crepitations dizziness. There is no history of head injury. These BP 180/62 mmHg headaches occur throughout the day. There have been several episodes of vomiting. In the past she has been fit (5) JVP Normal and well. Her mother and sister have both suffered Apex ‘Heaving’in character and episodes of migraine since childhood. She takes the oral displaced laterally contraceptive pill as her only medication. On examina- Auscultation Grade 3 crescendo-decrescendo tion she is obese and has a BMI of 32. She is normoten- systolic murmur over whole sive. Cranial examination is normal. Fundoscopy precordium,not heard in reveals bilateral blurring of the optic disc margins. carotid area There are no abnormal findings in the peripheral nerv- Chest Few bi-basal crepitations ous system. BP 110/93 mmHg CUM1 2/15/05 4:33 PM Page 7

Paper 1 EMQs 7 USIN 1 QUESTIONS (3) A 52-year-old patient attends complaining of (e) Giant cell arteritis episodic bouts of . He has no previous history (f) Space occupying lesion of headaches. The pain is described as burning and se- (g) Chronic subdural haematoma vere.The pain occurs behind the right eye, and is associ- (h) Epilepsy ated with lacrimation, and running of the nostril on the (i) Cavernous sinus thrombosis same side.Each episode lasts approximately 1 h and then (j) Viral meningitis subsides. In the past he has suffered from diabetes and (k) Benign intracranial hypertension . He smokes 20 cigarettes per day. His current (l) Cluster headache medication includes metfomin and GTN spray for occa- (m) sional use. You happen to see him during an acute (n) Hypertensive encephalopathy episode. He has conjunctival injection on the side of the (o) Acute encephalitis pain. Cranial nerve examination and fundoscopy find- (p) None of these ings are normal. Question 3 (4) A 58-year-old female patient presents with her rela- tives with a history of increasing headaches. The pain is Match the following chest examination findings to the felt worse at the top of the head. These occur every day most likely diagnosis: and improve towards the end of the day.In addition there have been several reported episodes of irregular ‘jerking’ (1) Respiratory rate 36 b.p.m. of the left ,each lasting around 5min.There is no his- Central cyanosis Present tory of head injury.There is a 10-year history of excessive Trachea Central alcohol consumption, 30 units per day. She takes no Chest shape Normal regular medication. On examination she is apyrexial. Expansion Equal There is a bilateral coarse tremor. There is a single spider Percussion note Resonant bilaterally naevus on the upper chest wall, but no jaundice or he- Auscultation Normal breath sounds patomegaly. Cranial nerve examination reveals a right- sided homonymous inferior quadrantanopia. (2) Respiratory rate 32 b.p.m. Central cyanosis Present (5) A 19-year-old university student presents acutely Trachea Deviated to the right with a headache, vomiting and photophobia. He had Chest shape Normal been found confused, wandering around his hall of resi- Expansion Reduced on left dence looking for his room.He had been fit and well until Percussion note Hyperresonant on left a few hours ago.Prior to admission,in the ambulance,he Auscultation Reduced breath sounds on had been observed to have a grand mal fit, which has left responded to rectal diazepam. Past medical history includes an admission for observation 6 months previ- ously following a minor head injury after falling off his (3) Respiratory rate 16 b.p.m. bike. He had received the meningitis C vaccine before Central cyanosis Absent going to university. On examination he was pyrexial at Trachea Deviated to left side 38.7°C. Confused and GCS 10/15. There was obvious Chest shape Depressed chest wall at left stiffness and photophobia. There was no rash. Cra- apex nial nerve examination was normal. The pupil size was Expansion Reduced in left upper zone 3 mm bilaterally and reactive and the optic discs were Percussion note Dull over left clavicle thought to be normal. Auscultation Normal

(a) Trigeminal (b) Migraine (c) Acute bacterial meningitis (d) Tension headache CUM1 2/15/05 4:33 PM Page 8

8 Paper 1 EMQs

had run a cross-country race at school without prob- (4) Respiratory rate 20 b.p.m. lems. There is no family history of haemophilia or other Central cyanosis Absent bleeding disorder. On examination he was mildly pyrex- Trachea Deviated to the right ial at 37.5°C and he appeared in pain. The right knee was Chest shape Normal hot, swollen and locally tender. The following investiga- Expansion Reduced at right base QUESTIONS 1 tions were obtained: Percussion note Stony dull at right base Hb 12.3 g/dL Auscultation Reduced breath sounds at WCC 5.6 ¥ 109/L right base Platelets 354 ¥ 109/L PT 12 s (NR 10–15 s) (5) Respiratory rate 28 b.p.m. APPT 72 s (NR 25–35 s) Central cyanosis Present Bleeding time Normal Trachea Central Factor VIII:C 5% of normal Chest shape Normal Expansion Normal Percussion note Dull at right base (2) A 34-year-old man presents with a swollen and Auscultation Bronchial area at right base painful right knee. There is no preceeding history of trauma. His brother had had similar problems in the (6) Respiratory rate 28 b.p.m. past.The following investigations were obtained: Central cyanosis Present Hb 14.7 g/dL 9 Trachea Central WCC 10.6 ¥ 10 /L 9 Chest shape Hyperinflated bilateral Platelets 373 ¥ 10 /L Expansion Equal PT 14 s (NR 10–15 s) Percussion note Resonant equally left and APPT 79 s (NR 25–35 s) right sides Bleeding time Normal Auscultation Diffuse polyphonic Factor VIII:C Normal expiratory wheezing (3) A 27-year-old female is investigated for heavy peri- (a) Lung abscess ods. Her mother had similar problems. She had also no- (b) Right lower lobe pneumonia ticed prolonged bleeding from simple cuts and after (c) Right sided pleural effusion blood taking. She was on no medication. The following (d) Old tuberculosis at right apex investigations were obtained: (e) Acute pulmonary oedema Hb 10.2 g/dL 9 (f) Asthma WCC 9.6 ¥ 10 /L 9 (g) Tension pneumothorax on the right Platelets 175 ¥ 10 /L (h) Left ventricular failure PT 11 s (NR 10–15 s) (i) Left apical lung fibrosis APPT 79 s (NR 25–35 s) (j) Right ventricular failure Bleeding time 18 min (normal <10 min) (k) Pulmonary embolism Factor VIII:C 20% of normal (l) Inhaled foreign body in right main bronchus (m)None of these (4) A confused 78-year-old patient is admitted and noted to have numerous bruises. Her relatives say she takes a range of medications, but the details are not Question 4 known and no reliable history can be obtained from the From the following questions, choose the best diagnosis patient. In her bag is found a outpatients ap- from the list below. pointment card. On examination she is confused and drowsy with a GCS of 9/15. There are multiple large (1) A 7-year-old boy presents with spontaneous onset of bruises of the skin. She has an enlarged right pupil that swelling and pain, in his right knee. The previous day he reacts poorly to light.Her pulse is 52/min and irregularly CUM5 2/15/05 4:36 PM Page 49 ANSWERS 1 Paper 1 Answers

MCQs

[1] (a) F (b) F (c) T (d) F (e) F •Ischaemic myocardial disease, coronary artery [2] (a) F (b) F (c) F (d) F (e) F disease [3] (a) F (b) T (c) F (d) F (e) F •Cardiomyopathy (alcoholic, diabetic, drug- [4] (a) F (b) F (c) T (d) T (e) T induced,idiopathic) [5] (a) F (b) F (c) F (d) F (e) T •Myocarditis [6] (a) F (b) F (c) F (d) T (e) F •Valvular heart disease [7] (a) F (b) T (c) F (d) T (e) F •Congenital heart disease with severe pulmonary [8] (a) T (b) F (c) T (d) F (e) F hypertension [9] (a) T (b) F (c) T (d) F (e) T •Terminal ventricular septal defect or atrial septal [10] (a) F (b) F (c) T (d) T (e) F defect [11] (a) F (b) T (c) T (d) F (e) T • Dominant diastolic heart failure [12] (a) F (b) F (c) T (d) F (e) F •Hypertension [13] (a) F (b) F (c) F (d) T (e) F •Severe aortic stenosis [14] (a) F (b) T (c) F (d) F (e) T •Hypertrophic cardiomyopathy [15] (a) T (b) F (c) F (d) F (e) T •Restrictive cardiomyopathy [16] (a) F (b) T (c) F (d) T (e) T •Ischemic myocardial disease, coronary artery [17] (a) T (b) T (c) T (d) T (e) T disease [18] (a) F (b) F (c) T (d) F (e) F • Acute heart failure [19] (a) F (b) F (c) F (d) T (e) F •Acute mitral or aortic regurgitation [20] (a) F (b) F (c) F (d) F (e) F •Rupture of valve leaflets or supporting structures [21] (a) F (b) F (c) F (d) T (e) T •Infective with acute valve [22] (a) F (b) T (c) F (d) F (e) F incompetence [23] (a) F (b) F (c) T (d) T (e) T •Myocardial infarction [24] (a) F (b) F (c) F (d) F (e) F (b) Atrial cells store and release the 28-amino-acid atrial natriuretic peptide (ANP) in response to stretch of the right atriumby increased central venous pressure. The relative volume overload of the atria that occurs [1] (a) F (b) T (c) T (d) F (e) F in heart failure causes levels of ANP to be high, leading (a) Valvular disease only causes 7% of cases of heart to a variety of beneficial effects in heart failure,including failure in the UK. The commonest cause of heart failure ... in the UK is atherosclerotic ischaemic heart disease.In •Increased sodium and water excretion by the kidney general, the causes of low-output cardiac failure can be (natriuresis and diuresis) split up depending on whether they cause predominant- •Increased glomerular filtration, hence increased ly systolic,predominantly diastolic or acute failure of the sodium excretion (by selective vasodilatation of myocardium... renal glomerular afferents, and vasoconstriction of • Dominant systolic heart failure renal glomerular efferents)

49 CUM5 2/15/05 4:36 PM Page 50

50 Paper 1 MCQs

•Relaxation of vascular smooth muscle (vasorelaxa- tal convoluted tubule by binding to the chloride site of tion of capacitance vessels) the electroneutral Na+/Cl– co-transport system and in- •Increased vascular permeability hibiting its action. Potassium loss with these drugs is •Inhibition of release/action of several undesirable significant and can be serious. Excretion of uric acid hormones which exacerbate heart failure (e.g.aldos- (Æ gout) and calcium is decreased, whereas that of ANSWERS 1 ANSWERS terone,angiotensin II,endothelin,ADH) magnesium is increased. When it was first discovered, ANP was cast as the car- • Loop diuretics (e.g. furosemide, bumetanide): inhibit diovascular hero, standing opposed to the villainous in- transport of NaCl out of the lumen of the thick seg- tents of vasoconstrictors such as angiotensin, ADH and ment of the ascending limb of the loop of Henle.These endothelin in heart failure. Unfortunately, its potential are the most powerful of all currently used diuretics, therapeutic use foundered with the discovery of its short potentially causing loss of up to 25% of the Na+ in the plasma half-life. Subsequent attempts to devise ANP ag- filtrate by direct inhibition of the Na+/K+/2Cl– carrier onists or agents to block clearance of the endogenous in the luminal membrane.Again,these drugs cause sig- peptide have been thus far unsuccessful. nificant K+ loss. There is an increase in the excretion of (c) The third heart sound (S3) occurs in early diastole calcium and magnesium, and a decrease in the excre- due to rapid ventricular filling as soon as the mitral and tion of uric acid (Æ gout). tricuspid valves open. It can be normal in children and • Potassium-sparing diuretics (e.g. spironolactone, young adults, but is abnormal in others and represents amiloride): Spironolactone has a limited diuretic ac- heart failure or volume overload of the heart (e.g. mi- tion.By acting as an aldosterone antagonist it inhibits tral or aortic regurgitation).It is commonly referred to as Na+ retention and decreases K+ excretion. Similarly, a ‘distressed’sound. amiloride has limited diuretic efficacy.By blocking Na+ The (S4) occurs in late diastole in reabsorption in the collecting tubules and ducts, it association with ventricular filling due to atrial systole, concomitantly decreases K+ excretion. Importantly, and is related to reduced ventricular compliance.It is a drugs in this class are the only diuretics that do not low-frequency oscillation that can be normal at older cause hypokalaemia. ages owing to a physiological decline in ventricular com- pliance,but is nearly always abnormal at younger ages es- [2] (a) F (b) F (c) F (d) F (e) F pecially if it is of high intensity or is palpable. It is (a) Over 90% of all cases of hypertension arise due to no common in ventricular hypertrophy, particularly with known cause,referred to as ‘essential’ hypertension. hypertension and aortic stenosis, and is almost invari- The remaining cases of hypertension that do have a de- able in acute myocardial infarction.S4 may arise from the fined cause are known as ‘secondary’ hypertension. right ventricle, the left ventricle or both. It is commonly These are always important to consider because even referred to as a ‘stressed’sound. though they are rare, they are frequently amenable to (d) The chest X-ray in a patient with heart failure has a treatment. classical pattern,comprising... •Cardiomegaly (greater than the width of one Secondary causes of hypertension hemithorax) • Renal causes: these account for >80% of cases of sec- •Upper lobe venous diversion ondary hypertension.They cause inappropriate re- •Kerley-B lines (fine peripheral septal lines; named tention of salt and water and inappropriate elevation after Peter J. Kerley, an English radiologist who also of plasma renin levels.They may be split into: described Kerley-A and Kerley-C lines on chest radi- • Renovascular ographs) Renal artery stenosis •‘Bat’s wing’hilar oedema • Renoparenchymal •Bilateral effusions Chronic glomerulonephritis (e) Diuretics commonly used in the treatment of heart Chronic pyelonephritis failure include . . . Diabetic nephropathy • Thiazide diuretics (e.g. bendrofluazide, metolazone): Adult polycystic kidney disease decrease active reabsorption of Na+ and Cl– in the dis- Chronic tubulointerstitial nephritis CUM5 2/15/05 4:36 PM Page 51

Paper 1 MCQs 51

• Endocrine causes: there are eight... lessening chest pain in acute MI that they do in stable ANSWERS 1 •Conn’s syndrome (excess mineralocorticoid) Æ (b) angina, they are still able to noticeably lessen the pain of •Cushing’s syndrome (excess glucocorticoid) acute MI when given sublingually, buccally or intra- •Acromegaly (excess growth hormone) venously. •Adrenal hyperplasia (congenital) (b) So-called ‘silent’infarcts occur in the elderly,diabet- • Phaeochromocytoma ics and patients with a long history of hypertension. •Hyperthyroidism Instead of presenting with pain, these infarcts present •Hypothyroidism Æ (c) with dyspnoea from development of acute pulmonary •Hyperparathyroidism oedema, syncope or coma from dysrhythmias, acute • Cardiovascular causes:the most important is coarcta- confusional states, diabetic hypoglycaemic crisis, tion of the aorta (also polycythaemia rubra vera) hypotension or . • Pharmacological causes: including the oral contra- (c) Although administration of morphine (or more ceptive pill, corticosteroids, monoamine oxidase correctly diamorphine) is an important early manoeu- inhibitors (paroxysms when eating tyramine- vre in treating acute MI (since it produces analgesia and containing foods), cocaine, amphetamines, vaso- lessens subjective distress from dyspnoea), it should be pressin,etc. given intravenously (or orally if access is difficult) and • Pregnancy not intramuscularly. This is because should the patient • Raised intracranial pressure (ICP): an acute rise in later be thrombolysed, significant internal or external ICP causes the ‘Cushing phenomenon’ or ‘reflex’, bleeding can occur if intramuscular injections have been which is essentially a rise in systemic blood pressure in recently given. response to the increased intracranial pressure. Com- (d) Myocardial infarction should be suspected on the monly occurs in head injury or intracranial haemor- history,but is should only be diagnosed in the presence of rhage Æ (d) either positive ECG findings, or a raised level of cardiac To impress examiners with your knowledge of second- enzymes on testing the blood an appropriate interval ary causes of hypertension, also mention GRA (gluco- after onset of symptoms. corticoid remediable aldosteronism).In this condition (e) The first change of acute MI visible on the ECG is a there is a crossover mutation and fusion of the adreno- point of contention, although it most certainly is not the corticotrophic hormone (ACTH)-regulatory element of appearance of Q-waves. the 11-b-hydroxylase gene and the aldosterone synthase ST-segment elevation is said to occur within minutes gene, meaning that every time the body attempts to of the onset of acute MI and can persist for up to 2 weeks. manufacture steroid, it instead produces aldosterone, If elevation persists over 4 weeks,this suggests formation causing profound hypertension via salt and water reten- of a ventricular aneurysm. tion. Once discovered, the treatment is simple: give ex- Occasionally T-wave inversion is said to be the first ogenous dexamethasone to switch off the need for visible ECG change of acute MI. This tends to persist endogenous steroid production, thereby switching off longer than ST-elevation,yet it is still usually only a tran- inappropriate aldosterone production. sient change. T-wave inversion that occurs without (e) Malignant hypertension is said to occur when the BP subsequent formation of Q-waves is typical of sub-endo- rises suddenly and precipitously to levels above cardial infarction (non-Q-wave infarction). 140 mmHg diastolic. There is characteristic fibrinoid The final ECG change associated with acute MI is the necrosis of vessel walls and rapid progression to renal appearance of pathological Q-waves.These are broad failure. There are marked changes in retinal vessels, and (>1mm), deep (>2mm) negative deflections that start grades 3 and 4 on the Keith–Wagener classification of the QRS complex. They occur physiologically in aVR, I hypertensive retinopathy are diagnostic of malignant and III. Pathological Q-waves develop over hours or hypertension, but only in the presence of a diastolic days,and are the hallmark of transmural infarct.They re- BP > 140 mmHg. flect electrical silence of infarcted cardiac tissue, causing a window through which the normal endo- to epicardial [3] (a) F (b) T (c) F (d) F (e) F activation of the opposite,non-infarcted ventricular wall (a) Although nitrates do not have the marked effect of is ‘seen’.They are almost always permanent. CUM5 2/15/05 4:36 PM Page 52

52 Paper 1 MCQs

[4] (a) F (b) F (c) T (d) T (e) T or or posterior MI; treated first with atropine, then The complications of myocardial infarction can be with electrical pacing thought of under five main headings... • Atrioventricular blockade is also common; if 1. Further chest pain the infarction is an inferior one, the blockade is •A bruised sensation and musculoskeletal pain are often temporary and does not require treatment; ANSWERS 1 ANSWERS common within the first 48 h, especially if the pa- however, if the infarction is an anterior one and tient has undergone CPR or repeated attempts at 2nd or 3rd degree block/bifascicular block arises, DC cardioversion then prophylactic pacing is indicated • Pericarditis may develop in the first 1–3 days post- • Ventricular ectopics post-MI have a poor progno- MI; it is commoner with full thickness infarcts and sis as they frequently herald the development of may cause an audible rub (‘like walking in fresh- ventricular or fibrillation; there is no laid snow’); treat with high dose aspirin evidence that treatment of these alters prognosis, + 2+ • Infarct extension may occur; look for further ST- but it may be prudent to correct PaO2,K and Mg ; elevation; treat with repeated thrombolysis or ur- if ectopics continue, a magnesium sulphate infu- gent coronary angioplasty sion may be set up •It may simply be post-infarction angina;this usual- • Ventricular tachycardia may arise,and it should be ly develops within 10 days of the acute episode and treated in the usual way (Mg2+, lidocaine, amio- can be treated with standard medical therapy, i.e. darone, synchronized DC shock). VT in the first nitrates,b-blockers and Ca2+-channel antagonists 24 h post-MI has a less sinister prognosis than VT 2. Fever arising later in the post-MI course. Should ventric- •Fever commonly peaks 3–4 days post-MI and is due ular fibrillation develop from VT, prompt DC car- to myocardial necrosis dioversion should be instigated •Other causes of fever must be considered,e.g.infec- 5. Hypotension and shock post-MI tion, thrombophlebitis, venous thrombosis, drug •Common in large MIs; treatment is with cau- reaction, pericarditis,etc. tious plasma volume expansion (i.e. 100–200mL • Dressler’s syndrome occurs weeks or even months colloid over 10min), and then inotropes if the after MI and consists of the triad of pericarditis, BP should remain low despite adequate filling fever and ; it is actually an au- pressures toimmune response to the damaged myocardium; Other complications of MI worth considering are it may necessitate administration of anti-inflam- thromboembolism secondary to prolonged inactivity matories.Æ (c) or cardiac mural , left ventricular aneurysm 3. New systolic murmur (four to consider) formation (think persistent ST-elevation), heart failure • secondary to inflamma- and,of course,death. tion of the infarcted myocardium • Sheehan’s syndrome is pituitary necrosis due to circu- • Long-standing murmurthat was missed at presen- latory collapse following severe post-partum bleeds, tation and is nothing to do with MI Æ (a) • Ventricular septal defect: this classically occurs •Acute aortic regurgitation is not a complication of 5–10 days post-MI and presents as sudden collapse, acute MI. It happens more often in dissecting aortic pulmonary oedema and hypotension; usually diag- aneurysms or with valve destruction associated with nosed on echo,where colour flow shows left to right the vegetations of bacterial endocarditis Æ (b) flow across the septum; requires emergent surgical • Electromechanical dissociation (EMD) occurs when repair the ECG is relatively normal and yet there is no • Acute mitral regurgitation occurs 2–10 days post- mechanical activity of the heart (normal ECG but no MI due to infarction and rupture of papillary mus- output) Æ (d) cle; can cause torrential regurgitation and sudden The causes of EMD can be grouped into the 4 Hs and the death, but if the patient survives, emergent surgical 4 Ts ... repair is vital H Hypothermia T Tension pneumothorax 4. Post-MI arrhythmias (four to consider) H Hypoxia T Thromboembolism • Sinus is common, especially in inferi- (pulmonary) CUM5 2/15/05 4:36 PM Page 53

Paper 1 MCQs 53

H Hypovolaemia T Tamponade (cardiac) consumption by the difference in oxygen content ANSWERS 1 H Hypo- and T Toxins (i.e.drugs) of arterial and mixed venous blood. It is commonly hyperkalaemia used in operating theatres to monitor cardiovascular parameters. [5] (a) F (b) F (c) F (d) F (e) T (b) Claudication means limping,being derived from (a) Inferior infarcts cause ST-segment elevation in the Latin claudicatio,to limp. leads II,III and aVF.Remember:infarction causes eleva- (c) Leriche syndrome is aortoiliac occlusive disease, tion, ischaemia causes depression. Inferior infarcts usu- which causes absent femoral , lower extremity ally reflect occlusion in the RIGHT coronary artery or claudication,wasting of buttock muscles and impotence. one of its branches. It can be caused acutely by a ‘saddle embolism’ at the bi- (b) Pericarditis causes ‘saddle-shaped’ ST-segment el- furcation of the aorta, but is more commonly seen in the evation in all leads of the ECG. Downward-sloping ST- chronic setting of slowly progressive atherosclerosis. segment depression indicates myocardial ischaemia, (d) Rest pain indicates advanced peripheral vascular whereas upward sloping ST-segment depression is non- disease. It occurs most commonly in the toes and specific. metatarsal heads on lying down at night.Temporary re- (c) Atrial fibrillation causes the ECG to show fine oscil- lief can be obtained by dangling the legs over the side of lations of the baseline (F-waves) and no clear P waves at the bed, or by standing up and walking, because this all.The QRS segments occur in an irregularly irregular makes the feet dependent, increasing gravitational hy- fashion, and at a rapid rate. The causes of AF can be re- drostatic pressure, increasing venous pressure and so called by remembering that PIRATES were often dehy- temporarily enhancing oxygen delivery.Unlike claudica- drated after long spells at sea... tion, which is caused by muscle ischaemia, rest pain is Dehydration caused by nerve ischaemia, and it often corresponds to P Pulmonary disease an ankle:brachial pressure index (ABPI) of <0.4.It I Ischaemia,infarction should be differentiated from benign nocturnal cramps, R Rheumatic heart disease which usually occur in the calf and are not associated A Anaemia,atrial myxoma with impaired blood flow. T Thyrotoxicosis (e) Arterial ulcersarise due to arterial insufficiency.They E Ethanol tend to occur on the toes, heel or dorsum of the foot in S Sepsis response to minor trauma and are exquisitely painful. (d) First-degree heart block is manifest on the ECG as They have a punched-out appearance and a pale simple prolongation of the PR-interval.In health, the necrotic base. PR-interval should be between three and five small Venous ulcers usually occur at the medial or squares on the ECG,i.e.0.12–0.2s.If it is longer than this, lateral malleolus in response to venous pooling. They then first-degree heart block is present. NB In first- often cause a brownish discoloration of the skin degree heart block, the PR interval is constant even (haemosiderin deposition),have a granulating base and though it is prolonged,and a QRS complex follows every whilst painful,do not cause anywhere near as much pain P-wave.Broad QRS complexes suggest a rhythm of as arterial ulcers. They are associated with significant ventricular origin OR a supraventricular rhythm with oedema.Treatment is usually conservative. concomitant bundle branch block. Neuropathic ulcersare painless,and usually located on (e) Ventricular aneurysm characteristically produces the plantar or lateral aspects of the foot.They are the di- persistent ST-segment elevation in all leads for greater rect result of diabetic neuropathy and are often asso- than 1 month’s duration. ciated with destruction of the ankle joint (Charcot’s foot).Because of the nature of diabetes,these ulcers often [6] (a) F (b) F (c) F (d) T (e) F co-exist with arterial ulceration. (a) Flow rate within a vessel is, in fact, governed by Poiseuille’s law,which states that the flow within [7] (a) F (b) T (c) F (d) T (e) F a vessel is proportional to the fourth power of the (a) Aortic stenosis is heard best in the aortic area (2nd radius. The Fick principle is a means of indirectly right intercostal space) with the patient leaning forward calculating cardiac output by dividing total body oxygen and the breath held in end-expiration (‘breathe in, CUM5 2/15/05 4:36 PM Page 54

54 Paper 1 MCQs

breathe out, hold it’). Aortic regurgitation is best heard (b) Thyroid eye signs (i.e. retro-orbital swelling, prop- at the left sternal border with the patient sitting forward tosis,exophthalmos,limitation of eye movements,visual in end expiration. impairment due to optic nerve pressure) are only seen (b) Aortic stenosis causes isolated hypertrophy of the in Graves’ disease.All causes of hyperthyroidism, left ventricle. The left atrium does not become enlarged however, may cause atrial fibrillation, because it is the ANSWERS 1 ANSWERS or hypertrophied because the mitral valve is intact (un- high level of circulating thyroid hormone and not the less there is mixed valvular disease). high levels of IgG (thyroid stimulating antibody) seen (c) A water-hammer pulse describes a pulse with a in Graves’ that is responsible for precipitating atrial forcible impulse but immediate collapse,and is charac- fibrillation. teristic of aortic regurgitation. It is also known as a col- (c) Because there is no ordered contraction of the atria lapsing pulse. The pulse character associated with aortic in AF (i.e. electrical activity does not start at and is not stenosis is slow-rising because of the resistance encoun- propagated from the sino-atrial node), there are no P- tered by the left ventricle when pushing blood past the waves visible on the ECG. stenosed valve. (d) Anti-dysrhythmic agents are traditionally classified The character of pulsation may also be described in into four groups according to the Vaughan-Williams the following ways... classification ... • Pulsus paradoxus: an exaggeration of the normal ten- •Class 1: Membrane depressants dency of the systolic pressure to fall on inspiration •Class 1a: disopyramide,quinidine (should be <10mmHg in health,but is greater than this •Class 1b: lidocaine in pulsus paradoxus); characteristically seen in severe •Class 1c: flecainide,propafenone airway obstruction (e.g.acute severe asthma),cardiac •Class 2:Antisympathetics tamponade and constrictive pericarditis. e.g. b-blockers, the cardioselective (i.e. b-1 specific) • Pulsus alternans: alternating weak then strong (but ones being metoprolol and atenolol regular) beats; characteristic of severe myocardial •Class 3: Prolongers of the action potential failure,indicating very poor prognosis. e.g.amiodarone,sotalol • : ectopic beats follow each sinus •Class 4: Slowers of conduction in nodal tissue beat,causing beats to occur in pairs; rhythm is not reg- e.g. the non-dihydropyridine calcium channel ular. antagonists—verapamil,diltiazem • :an arterial pulse with palpable,sepa- Class 1c agents (like flecainide) and all other class 1 anti- rated peaks;characteristic of hypertrophic, obstruc- dysrhythmics should only really be used in the treatment tive cardiomyopathy (HOCM) and when aortic of intractable or life-threatening arrhythmias;they stenosis co-exists with aortic regurgitation. should never be used where the patient has significant left (d) The murmur caused by aortic stenosis is a diamond- ventricular dysfunction (e.g. heart failure), prior history shaped ejection systolic murmur,best heard in the aor- of MI or acute coronary ischaemia. tic area. It is said to be rough in quality, and is longer the (e) Treatment of AF can be approached in two ways: more severe the stenosis. It radiates to the carotids. 1. Control the ventricular rate and ignore what the A systolic ejection click may be heard when the valve atria are doing: this is usually achieved using an AV- opens if it is calcified and has become immobile (severe nodal blocking drug (e.g. digoxin, b-blockers or disease). There may be a prominent fourth heart sound verapamil). (S4) if the left ventricle is hypertrophied and has become 2. Cardiovert in an attempt to stop the atria fibrillating: stiff. this can be performed electrically (80% of patients (e) Aortic stenosis alone is not associated with the devel- convert to sinus rhythm with synchronized DC car- opment of AF as long as the mitral valve remains intact. dioversion) or medically (with an IV infusion of class 1a, 1c or 3 drug, usually amiodarone). Recurrent [8] (a) T (b) F (c) T (d) F (e) F paroxysms can be prevented with prophylactic oral treat- (a) Dehydration and hypovolaemia, whether acute or ment with a class 1a, 1c or 3 drug, again usually chronic, are common causes of atrial fibrillation, espe- amiodarone. cially in hospital patients. Amiodarone is one of a select number of drugs whose CUM5 2/15/05 4:36 PM Page 55

Paper 1 MCQs 55

side effect profile should be known for undergraduate of hydrogen ions in cerebrospinal fluid (CSF), stimulat- ANSWERS 1 final examinations... ing the respiratory centre. Sensitivity to in-

•Sensitivity to sunlight (i.e.phototoxicity) creasing PaCO2 can be lost in chronic obstructive •Corneal microdeposits pulmonary disease (COPD) due to chronic over-expo-

•Slate-grey skin discolouration sure of the brainstem to high levels of CO2, and in these •Peripheral neuropathy patients the chief stimulus to breathe becomes the partial

•Hypo- or hyperthyroidism pressure of oxygen (PaO2),as detected by the carotid and •Pulmonary fibrosis (irreversible) aortic arch bodies. Patients with chronic type II respira- tory failure should therefore not be administered high [9] (a) T (b) F (c) T (d) F (e) T concentrations of oxygen because it abolishes their respi- (a) The right main bronchus is wider,shorter and more ratory stimulus,and they would quickly have a respirato- vertical than the left; this is why a swallowed foreign ob- ry arrest. Pyrexia, large doses of aspirin and the drug ject or aspirated material is more likely to impact in the doxapram may directly stimulate the brainstem respira- lower lobe of the right lung rather than the left. Material tory centre, whereas severe hypoxaemia and sedatives, aspirated when a patient is lying flat is most likely to im- especially opioids,may depress it. pact in the middle lobe of the right lung. (b) The lung has a dual blood supply.The bronchi, [10] (a) F (b) F (c) T (d) T (e) F connective tissue of the lung and the visceral pleura On examination of the chest in a patient with a unilateral receive their blood supply from the bronchial tension pneumothorax... ,which are branches of the descending aorta.The •Chest wall movements will be decreased on the alveoli on the other hand receive deoxygenated affected side only Æ (a) blood from the terminal branches of the pulmonary •The mediastinum will be displaced away from the arteries. affected side Æ (b) (c) The epithelial lining of the alveoli is mainly made up •The percussion note will be hyperresonant on the of type 1 pneumocytes.These form a thin barrier for gas affected side Æ (c) exchange.They are, however, derived from surfactant- •Breath sounds will be decreased or absent on the affect- producing type 2 pneumocytes, which are slightly more ed side Æ (d) in number but cover less surface area. •Tactile vocal resonance will be decreased on the affect- (d) Both lungs have oblique fissures, but only the right ed side Æ (e) has a horizontal fissure,hence there are three lobes in •There are no added sounds the right lung (upper, middle and lower) but only two in It is very useful for exams to be exactly sure of the chest the left (upper, lower and the lingula — a redundant seg- signs of the most common respiratory complaints. Re- ment of pulmonary tissue anterior to the surface of the cent exams have contained MCQs and EMQs that tested heart). precisely the candidate’s knowledge of such signs.A use- (e) In health the strongest stimulation to ventilation is ful table to memorize is shown below,but there is no sub-

an increase in the arterial partial pressure of CO2 stitute for having seen/discovered such signs for yourself: (PaCO2), which causes an increase in the concentration find patients with signs and examine them!

Pathological Chest wall Mediastinal Percussion Vocal process movement displacement note Breath sounds resonance Added sounds

Consolidation Decreased None Dull Bronchial Increased Fine creps ipsilaterally

Collapse Decreased Towards Dull Decreased or Decreased None ipsilaterally absent or absent Continued on p. 56 CUM5 2/15/05 4:36 PM Page 56

56 Paper 1 MCQs

Pathological Chest wall Mediastinal Percussion Vocal process movement displacement note Breath sounds resonance Added sounds

Fibrosis (general) Decreased None Normal Vesicular Increased Fine creps

ANSWERS 1 ANSWERS bilaterally

Pleural effusion Decreased Away Stony dull Vesicular Decreased None (>500 mL) ipsilaterally (decreased or or absent absent)

Large Decreased Away Hyper- Decreased or Decreased None pneumothorax ipsilaterally resonant absent or absent

Asthma Decreased None Normal Vesicular,with Normal Expiratory bilaterally prolonged polyphonic expiration wheeze

COPD Decreased None Normal Vesicular,with Normal Coarse creps bilaterally prolonged plus expiratory expiration polyphonic wheeze

11] (a) F (b) T (c) T (d) F (e) T coil of the lungs, decreased gas transfer, expiratory air- (a) COPD describes airways obstruction that occurs flow limitation and air-trapping. Whilst bronchitis and mainly in smokers or ex-smokers.The clinical distinc- emphysema are described as two separate clinical enti- tion between COPD and asthma is blurred, because vir- ties,they usually co-exist in all patients with COPD. tually all patients with COPD have a reversible element to (c) Patients with COPD commonly demonstrate aster- their disease, and is the reason these patients are treated ixis, which is a coarse, flapping tremor due to chronic re-

with bronchodilators [(b-adrenoceptor agonists such tention of CO2.It is also seen in patients with hepatic as salbutamol (Ventolin®) and antimuscarinics such as encephalopathy (‘liver flap’), although it can occur in a ipratropium bromide (Atrovent®)] and corticosteroids wide variety of metabolic and toxic encephalopathies.

(initially prednisolone orally, and if this is successful, CO2 retention also causes the hands to be warm (vasodi- wean to inhaled beclamethasone). latation) and the pulse to be bounding (hyperdynamic (b) Chronic bronchitis is defined on the basis of the his- circulation). toryas ‘cough productive of sputum on most days for at (d) Cor pulmonale is the term used to describe the least 3months of the year,for more than 1year’.This re- process of adaptation and failure the right side of the flects the nature of the underlying condition: hypertro- heart undergoes as a result of lung disease.Normally phy of the mucous-secreting glands of the bronchial tree the pulmonary vasculature responds to local hypoxia by (mainly large bronchi).Emphysema,however,is defined arteriolar constriction. So, for example, in lobar pneu- pathologically as ‘dilatation and destruction of the monia, blood flow to the affected lobe is decreased, mi- lung tissue distal to the terminal bronchioles’.Thus, it nimizing the amount of poorly oxygenated blood affects only the small airways,leading to loss of elastic re- reaching the systemic circulation. When the pneumonia CUM5 2/15/05 4:36 PM Page 57

Paper 1 MCQs 57

resolves,the vasoconstriction ceases and the flow returns called non-cardiogenic pulmonary oedema,where the ANSWERS 1 to normal. However, if hypoxia is widespread and irre- typical features are no upper lobe venous distension, no versible, as is the case in COPD, the resultant arteriolar septal lines or pleural effusion, a normal heart size and constriction is equally widespread and irreversible, thus the most reliable sign,which is peripheral alveolar shad- causing pulmonary hypertension. Over time the right owing and/or an air bronchogram.The CXR in some- atrium and ventricle hypertrophy,eventually the limit of one with suspected respiratory failure should also be the heart’s ability to adapt is reached, and the right heart checked carefully for pneumothorax, consolidation, fails, leading to the typical of right- bronchogenic cancer and oligaemia or wedge-shaped in- sided cardiac failure. farcts (i.e. signs of PE). Localized oligaemia on CXR is (e) Respiratory failure occurs when pulmonary gas ex- also known as ‘Westermark’s sign’,and is usually indica- change is sufficiently impaired to cause hypoxaemia tive of PE.

(PaO2 < 8kPa) with or without hypercapnia The main causes of respiratory failure can be remem- (PaCO2 > 7kPa).There are two types of respiratory bered if you remember that A DIMPLE can cause res- failure. piratory failure ... • Type I respiratory failure — ‘acute hypoxaemic’ A Asthma,ARDS,aspirin • mainly due to ventilation–perfusion mismatch D Drugs (opiates)

•PaO2 low (<8kPa) I Infection (pneumonia) •PaCO2 normal or low (<5kPa) M Metabolic acidosis (e.g.diabetic ketoacidosis) •Common causes include pulmonary oedema,pneu- P PE,pneumothorax monia,ARDS, (PE) L Left ventricular failure • Type II respiratory failure — ‘ventilatory failure’ E Effusions

•PaO2 low (<8kPa) •PaCO2 high (>7kPa) [13] (a) F (b) F (c) F (d) T (e) F •Occurs when alveolar ventilation is insufficient to (a) Asthma is characterized by chronic inflammation of

excrete the volume of CO2 being produced by tissue the airways which normally presents with a classical triad metabolism of symptoms... •Due to reduced ventilatory effort, inability to over- 1. Wheeze come increased resistance, failure to compensate for 2. Nocturnal cough

increased dead space or CO2 production,or a combi- 3. Dyspnoea nation of these Within the airways themselves,it has three main char- •Most common cause is COPD acteristics: bronchial inflammation (mainly involving •Other causes include chest wall deformity,respirato- eosinophils and mast cells with associated plasma exuda- ry muscle weakness [e.g. Guillain–Barré syndrome, tion, oedema, smooth muscle hypertrophy and mucus motor neurone disease (MND)] and depression of plugging), airways hyper-responsiveness and re- the respiratory centre (e.g.opioid excess) versible airflow limitation.It is important to realize, however, that with chronic asthma, the airflow limita- [12] (a) F (b) F (c) T (d) F (e) F tion does become irreversible to a certain degree.It

(a) PaCO2 is not elevated in type I respiratory failure. is also the case that aspects of the acute severe asthma (b) The commonest cause of type II respiratory failure is that continues to kill young people are also irreversible COPD. Pneumonia tends to cause type I respiratory (otherwise no-one would die from asthma as long as failure. they were treated with the correct medications). (c) Pulsus paradoxus is explained in the answers to (b) Typical asthma attacks occur after the conclusion MCQ [7],above. of,and not during,exercise.Cold,dry air cools and dries (d) Respiratory failure does not always present with the epithelial lining of the bronchi,precipitating attacks. dyspnoea,e.g. opioid overdosage, Guillain–Barré syn- (c) ACE-inhibitors are safe to give to asthmatic patients.

drome. Chronic retainers of CO2 may also present ACE-inhibitors should never,however,be given to pa- drowsy and not obviously dyspnoeic. tients with renal artery stenosis as they block produc- (e) The CXR described in the question is one of classical tion of angiotensin II and aldosterone, leading to a heart failure. The CXR in respiratory failure is one of so- decrease in the already threatened renal perfusion, and CUM5 2/15/05 4:36 PM Page 64

64 Paper 1 EMQs

[24] (a) F (b) F (c) F (d) F (e) F (e) In paralytic ileus, there is characteristically no phys- (a) The commonest cause of small bowel obstruction ical obstructionto passage of the luminal contents of the worldwide is herniae,whilst the most common cause in bowel , i.e. it is functional.Causes of paralytic ileus industrialized countries is postoperative adhesions. include . . . (b) This is the description of an intussusception.A •Sepsis (either within or adjacent to the peritoneal ANSWERS 1 ANSWERS volvulus occurs when a length of large bowel becomes cavity,e.g.pneumonia) twisted on itself, using the mesentery as the axis about •Drugs (e.g.opiates) which it twists. •Retroperitoneal haematoma (c) In ileus, a large (>2.5cm) stone passes • Electrolyte abnormalities (e.g.hypokalaemia) from or common bile duct into the •Physical inactivity GIT through a fistula.The stones usually lodge in the • Laparotomy terminal ileum. Paralytic ileus classically demonstrates absent bowel (d) Bowel sounds are characteristically infrequent, sounds,and may be definitively diagnosed from physical high-pitched and ‘tinkling’ in small bowel obstruction. obstruction by contrast studies and a history of less ab- They would be absent in paralytic ileus. dominal pain.

EMQs

These findings will also be seen in pulmonary hyper- Answer 1 tension due to other causes (e.g. congestive cardiac fail- (1) Mitral stenosis with atrial fibrillation ure,COPD,chronic lung pathology,primary pulmonary On examination the patient may have a malar flush and hypertension). be in atrial fibrillation.There may be evidence of a cere- brovascular accident (hemiplegia, facial weakness). An Causes of mitral stenosis opening snap is followed by a ‘rumbling’mid-diastolic • (a common cause) murmur,localizing (often very localized) over the ‘tap- •Congenital ping’ (palpable first heart sound). Listen with •Calcification the patient in the left lateral position with the bell of the stethoscope in several positions over the apex.If difficult to hear, the murmur may be accentuated by asking the Complications (often the patient is patient (if this is possible) to sit up and lie down five times asymptomatic) so raising the cardiac output.There is pre-systolic accen- •Recurrent chest infections (wet lungs) tuation only if the heart is in sinus rhythm and is due to •Atrial fibrillation and embolization (stroke) atrial systole. •Pulmonary hypertension and pulmonary oedema In addition if pulmonary hypertension has devel- •Right ventricular failure oped there may be a... •Tricuspid regurgitation • Left para-sternal heave (right ventricular •Bacterial endocarditis hypertrophy) • Loud second heart sound over the pulmonary area The severity of the mitral stenosis is •Possible murmur of pulmonary regurgitation (Gra- indicated by: ham Steell) •Signs of pulmonary hypertension (see above) •Murmur oftricuspid regurgitation [systolic murmur •Close proximity of the opening snap after the second at lower left sternal edge plus large ‘V’waves in the JVP heart sound (jugular venous pulse)] •The length of the mid-diastolic murmur (directly pro- • Right ventricular failure (elevated JVP, he- portional, however, very tight lesions lead to quiet patomegaly,ascites and peripheral oedema) murmurs) CUM5 2/15/05 4:36 PM Page 65

Paper 1 EMQs 65

ECG findings There may be a third heart sound (volume overload) ANSWERS 1 •Atrial fibrillation and a diastolic flow murmur due to the re-entry of the re- •P mitrale (bifid ‘M’shaped P waves best seen in lead II) gurgitated blood back into the left ventricle. Clinically the main differential will be aortic stenosis CXR appearances causing an ejection systolic murmur, a ventricular septal •Prominent left atrium may be the only abnormality defect or tricuspid regurgitation which both cause pan- •Calcification of the valve ring systolic murmurs. •Possible features of congestive cardiac failure Tricuspid regurgitation,however,will result in promi- •Kerley-B lines nent V waves in the JVP,a murmur that increases during •Upper lobe venous diversion inspiration and possible hepatic pulsations. •Bilateral effusions •Cardiomegaly Causes of mitral regurgitation •‘Bat’s wing’oedema spreading from the hilar areas •Myocardial infarction (acute presentation following an acute MI due to rupture of the chordae tendineae) Management •Rheumatic fever (acute or old) •Diuretics for symptoms of breathlessness •Left ventricular dilatation •Anticoagulation and digoxin if atrial fibrillation •Severe hypertension develops •Left ventricular failure •Prophylactic antibiotics for ‘dirty’ procedures to pre- •Cardiomyopathy vent valve infection (dental procedures or pelvic sur- •Bacterial endocarditis gery). Check the British National Formulary for the •Connective tissue diseases current recommendations • SLE •Surgical relief of stenosis if secondary pulmonary hy- • Elastic tissue disorders pertension develops •Marfan’s syndrome •Open surgery can be avoided in those unfit for a • Ehlers–Danlos syndrome general anaesthetic by percutaneous trans-septal bal- •Prolapsing mitral valve; ‘floppy mitral valve’.This oc- loon valvotomy. Here a balloon catheter is passed via a curs in young women. Embolic events and atypical peripheral into the right atrium. The atrial sep- chest pain (localized over the apex) may occur.A mid- tum is then perforated and the balloon is passed across systolic click is heard. the mitral valve and inflated.Mitral regurgitation is an The severity of mitral regurgitation can be assessed by inevitable consequence. Patients most suitable for this the presence of a . . . approach include those with: •Third heart sound •Minimal mitral regurgitation •Mid-diastolic flow murmur •Mobile valves (no calcification) •Left ventricular enlargement •Minimal subvalvular disease •Thrill Patients more suitable for formal valve replacement in- clude those with: (3) Mitral stenosis •Significant mitral regurgitation Here the patient is not in atrial fibrillation so there is pre- •Extensive valve calcification systolic accentuation of the murmur. •Thrombus in the left atrium (4) Aortic regurgitation with left ventricular (2) Mitral regurgitation failure The patient may be in atrial fibrillation(left atrial dilata- The patient has a wide pulse pressure on measuring the tion). The JVP may be elevated if associated with right blood pressure. The pulse is collapsing when the left ventricular failure (secondary to pulmonary hyperten- brachial artery palpated with arm elevated. The apex sion). The apex is displaced and ‘thrusting’ (a dynamic beat is displaced (state the location) and is thrusting in low-pressure movement indicating volume overload). character,indicating volume overload. The first heart sound is soft.There is a pan-systolic With the patient sat forward and the breath held in murmur heard at the apex radiating to the left axilla. end expiration,a short early decrescendo murmur radi- CUM5 2/15/05 4:36 PM Page 66

66 Paper 1 EMQs

ating down the left sternal edge is heard (listen in to replace the valve before irretrievable ventricular dys- several positions down the left sternal edge with the di- function occurs as indicated by an enlarging heart on aphragm). The longer and louder the murmur, the echocardiography. Once the end systolic diameter of more severe the regurgitation. the left ventricle has reached 55 mm, then surgery is In this case, left ventricular failure is suggested by the recommended. ANSWERS 1 ANSWERS elevated JVP, displaced apex and bi-basal crepitations. There may be an additional ejection systolic murmur in (5) Aortic stenosis the aortic area due to increased flow across the aortic This patient has a slow rising pulse and a blood pressure valve. The aortic regurgitant jet can cause interference that has a narrow pulse pressure.The apex is displaced with the anterior mitral valve cusp resulting in a mid- and heaving indicating pressure overload. There may be diastolic murmur to be heard at the apex (Austin Flint a thrill felt with the ulnar border of the palm over the aor- murmur). tic area (indicating at least a grade 4 murmur and often Inspect the patient for Marfan’s body habitus, anky- reflects a pressure gradient across the valve of at least losing spondylitis (kyphosis of the spine) features of 40 mmHg). An ejection click may be heard over the left rheumatoid arthritis and the small, irregular Argyll sternal border indicating the stenosis is occurring from Robertson pupils (which react to accommodation but the valves (and not subvalvular or supravalvular in not to light) of syphilis. origin) and is due to opening of the valve.A click is espe- cially heard in bicuspid aortic stenosis. Note that clicks Causes of aortic valve regurgitation may also be heard in pulmonary stenosis or with a pro- •Aortic root dilatation lapsing mitral valve. •Severe hypertension An ejection systolic (rough, sawing, diamond •Connective tissue disorders: shaped) murmur is heard all over the precordium, but Marfan’s syndrome especially in the aortic area and possibly radiating to Rheumatoid arthritis the carotids. The second heart sound is quiet.The Ankylosing spondylitis differential diagnosis includes mitral regurgitation, • hypertrophic obstructive cardiomyopathy and aortic •Dissection of the aorta sclerosis. •Syphilis Aortic sclerosis is simply a noisy valve due to calcifica- •Valve disruption tion frequently seen in the elderly, and has no abnormal •Congenital (e.g.bicuspid valves) increase of the pressure gradient across the valve.Thus in •Rupture of the sinus of valsalva aortic sclerosis the patient has a normal pulse character,a • Endocarditis normal pulse pressure (blood pressure) and a normal •Rheumatic fever apex beat position and character. •Ventricular septal defect (loss of support) Clinical features of aortic stenosis Associated eponymous signs • due to arrhythmias (ventricular) • Corrigan’s sign:vigorous arterial pulsations in •Syncope due to arrhythmias neck •Chest pain (angina) • De Musset’s sign: head nodding with arterial •Dyspnoea (left ventricular failure) pulsations •Sudden death (10–20%) • Quinke’s sign:visible nail bed capillary pulsations. In aortic stenosis the ventricular function is relatively Partially compress the nail bed and look for pulsations well preserved, unlike in aortic regurgitation. However in the subungual capillaries with significant stenosis exercise fails to improve cardiac • Durozier’s sign: partially compress the femoral artery output leading to ischaemia and cardiac arrhythmias. in the inguinal area with a digit and listen proximally For this reason exercise tolerance tests are avoided in for a regurgitant murmur as blood turbulently flows these patients.In patients with aortic stenosis symptoms back under the ‘stenosis’during diastole are a good indication of the disease severity. Patients with aortic regurgitation may have few symp- Aortic stenosis is especially prone to bacterial endo- toms until left ventricular failure occurs. It is important carditis as it is a high-pressure valve lesion. CUM5 2/15/05 4:36 PM Page 67

Paper 1 EMQs 67

Causes of aortic stenosis avoid using a leading question when asking about ex- ANSWERS 1 •Valvular acerbating factors •Rheumatic fever • Progressively getting worse •Calcification • No ‘days off ’:relentless daily symptoms suggest a pos- •Congenital (e.g.bicuspid valve) sible serious cause •Supravalvular • New onset in middle life (i.e. uncharacteristic, no pre- •William’s syndrome (supravalvular aortic dia- vious history of headaches) phragm,mental retardation and hypercalcaemia) • Risk factors for intracranial bleeding: e.g.anticoagu- •Subvalvular lation,alcoholism,old age •Diaphragm below the aortic valve • Trauma:recent falls/head injury (frequently the •Septal trauma is forgotten) •Hypertrophic obstructive cardiomyopathy Examination alarm features The severity of aortic stenosis is indicated by: •Any reduction in the Glasgow Coma Scale •A narrow pulse pressure on blood pressure recording • Confusion (reduction in abbreviated mental test scor- •Reverse splitting of the second heart sound (i.e. pul- ing) monary before aortic) • Focal neurology: motor/sensory/coordination/visual/ •A soft second heart sound meningeal irritation •A fourth heart sound (indicating ventricular stiffness • Seizures due to left ventricular Hypertrophy) •Purpuric rash •A thrill • Signs of meningeal irritation •A heaving apex beat • Fever The ECG may show signs of left ventricular hyper- • Papilloedema trophy (R wave in V6 plus S in V2 summate to more than • Severe hypertension 35 mm) plus signs of ventricular strain (ST depression The diagnosis of headache is made largely on the and T wave inversion) in the leads, which look at the left history.A good history will pick out those where there ventricle (leads I,AVL,II,V5 and V6). should be heightened concern. The chest radiographmay show post-stenotic dilata- tion.There may be calcification of the aortic valve. (1) Migraine Alternatively there may be feature of congestive cardiac The features of migraine may be ‘classical’in (only) 20%, failure. presenting with a prodrome of visual auras (e.g. flashing In the management,valve replacement is indicated lights or zig-zag lines due to cerebral vasospasm) often for all patients with symptoms and where the pressure taking a stereotypical pattern for a given patient. This is gradient across the valve exceeds 50 mmHg. Balloon followed by a severe unilateral headache with photopho- valvotomy is reserve for those unfit for surgery or as a bia and phonophobia (intolerance of noise).Nausea and temporary measure. Antibiotic prophylaxis is required abdominal pain may occur along with focal neurology, for surgery. e.g.dysphasia,hemiplegia,sensory symptoms. However, some of these features may be lacking in ‘common migraine’ and differentiation from tension Answer 2 headaches (the main differential) can be difficult. The approach to first-line treatment is with paracetamol or Know the features for patients with a serious cause for ibuprofen (or both) used together with an anti-emetic,as headaches: gastric stasis during migraine can impair absorption. Codeine is avoided due to its predisposition to cause History alarm features ‘’headache,in addition to the risk of dependen- • Morning headache: improving after getting up cy and side effects such as constipation and drowsiness. • Morning vomiting Asking about triggers (food, stress, menstruation) and • Pain exacerbated by coughing,straining or bending: avoidance of these is helpful. Note that there is often a note that this occurs in many headaches to a degree, so family history in migraine sufferers. CUM5 2/15/05 4:36 PM Page 68

68 Paper 1 EMQs

(2) Benign intracranial hypertension •Memory/intellect deterioration A classical history. The problem is a recirculation disor- •Primitive reflexes return (Plantar response, grasp der of CSF.There is no focal neurology and the CT scan is and suck reflex) normal without ventricular dilatation.Prominent papil- •Expressive dysphasia (Broca’s area) loedema is found on examination. Enlargement of the •Anosmia ANSWERS 1 ANSWERS blind spot on testing the visual fields occurs;if untreated, •Parietal lobe blindness can occur. A CT scan is required, however, to •Apraxias (learned movements). exclude mass lesions and other causes of raised intracra- •Asteriognosis (shape recognition) nial pressure. Treatment involves stopping any causative •Homonymous inferior quadrantanopias drugs (e.g. steroids or oral contraceptives), weight loss •Extinction (sensory inattention for stimuli on the af- and thiazide diuretics. Repeated lumbar puncture and fected side when both sides stimulated) shunt operations are second line therapy. •Temporal lobe •Receptive dysphasia (Wernicke’s area) (3) Cluster headache •Memory loss Typically a pattern of recurrent headaches for a few •Cortical deafness days or weeks followed by temporary remission occurs. •Homonymous superior quadrantanopias 20% of cases are chronic. The male:female ratio is 5:1. •Occipital lobe The exact cause is unknown; however, alcohol may pre- •Cortical blindness cipitate an attack. Sufferers are more commonly smok- •Homonymous hemianopias ers. The episodes may be brief (up to an hour) but are recurrent in bouts, often with periods of remission last- Causes of cerebral mass lesions ing several months. Severe pain occurs behind the eye.A •Tumour partial Horner’s syndrome may occur along with vaso- •Secondary > Primary lesions motor dysfunction causing lacrimation and nasal dis- •Malignant > Benign charge. The treatment is similar to migraine and 5HT •Abscess () antagonists (e.g. sumatriptan) are often •Pneumonia helpful. In addition 100% oxygen (which may abort •Skin sepsis attacks), verapamil, lithium or methysergide are • ENT sepsis alternatives. Treatment can be very difficult (like some •Haemorrhage migraine). •Extradural haemorrhage (4) Space occupying lesion •Subdural haemorrhage This is a worrying history. The progression of focal neu- •Subarachnoid haemorrhage rology and seizures suggest an expanding mass lesion. •Granulomatous The exact neurological presentation depends on the •Cerebral sarcoidosis location; however, common patents of presentation •Vascular include; •AV malformations •Features of rising intracranial pressure (headache, •Hydrocephalus vomiting) •Neurological deficit (see patterns below) (5) Acute bacterial meningitis •Epilepsy From the given history it may seem difficult to separate Oedema around the lesion may exaggerate the picture. viral from bacterial meningitis. However, the lack of any An urgent CT scan is indicated.Remember the approach viral prodromal illness and the importance of not miss- to neurology: ‘Where is the lesion? Then, what is ing acute bacterial meningitis,make this the best answer. the lesion?’ The main other differential diagnoses include encephali- tis, subarachnoid haemorrhage, severe migraine, cere- Localizing mass lesions bral abscess,subdural and extradural haemorrhage. •Frontal lobe Immediate management includes antibiotics,which •Personality change should not be delayed awaiting investigations or even ad- •Disinhibition mission to hospital as this is associated with an adverse CUM5 2/15/05 4:36 PM Page 69

Paper 1 EMQs 69

outcome.Penicillin V 1.2 g should be given immediately. • Focal neurological signs are possible. Infarction or ANSWERS 1 A CT scan should ideally be performed before a lumbar false localizing signs due to raised ICP (C III,CVI palsy puncture ideally to exclude raised intracranial pressure or hemi-paresis) and other differential diagnoses. If meningococcal dis- • Shock (septicaemic) ease is suspected a lumbar puncture is avoided due to the • Bleeding (due to DIC) e.g. bleeding from venepunc- high risk of coning because of the prevalence of associat- ture sites ed cerebral oedema. Blood cultures (several sets) should Note that many of these features may be absent in certain be taken. The organism can also be looked for on PCR of patients with meningitis and that the disease may present blood or CSF. General resuscitation measures (IV fluids in a non-acute and non-specific manner. and oxygen) are required for septic shock,as well as close monitoring and support usually in intensive care. De- Which blood tests? spite optimum management overall mortality is still FBC/U and Es/CRP/PT/APTT/FDPs (fibrin degrada- around 10%.Early recognition of symptoms is therefore tion products),blood cultures. vital. Which antibiotics? Presentation of acute bacterial meningitis Penicillin V (2.4 g i.v. or 1 g i.m. stat if no IV access) and • Headache (bursting,gradual or abrupt onset) Cefotaxime(2g/8h i.v.).Usually hospitals will have their • Pyrexia own policy. •Signs of meningeal irritation, e.g. neck stiffness, Kernig’s sign Why CT scan? • Photophobia (a non-specific feature for meningitis) •Helps exclude other causes,e.g.IC bleed/subarachnoid • Rash: A non-blanching purpuric rash due to dissemi- haemorrhage/cerebral abscess nated intravascular coagulation (DIC) occurs in •Ideally prior to lumbar puncture to exclude raised ICP meningococcal meningitis only.The lesions may occur anywhere, and the patient should be completely in- Lumbar puncture patterns spected as even a single lesion may be significant Normal opening pressure: <16 cm manometric height of • Global cerebral dysfunction (cerebral oedema, water. toxaemia): confusion/drowsiness/convulsions/vacant In general the following apply (for exam purposes): staring

Bacterial Viral TB Normal

Appearance Cloudy Clear Clear Clear Cells Neutrophils Lymphocytes Lymphocytes <5/mm3 Glucose <1/3 levels in blood Normal <1/2 blood levels 1/2–2/3 blood levels Protein >1 g/L <1 g/L >1 g/L 0.2–0.4 g/L

Answer 3 •Haemoptysis •Chest pain (pleuritic type; a late feature) (1) Pulmonary embolism •Sudden collapse (cardiac arrest) The diagnosis of pulmonary embolism relies on a com- bination of the following... An assessment of the risk factors for thromboembolism An assessment of the clinical symptoms • Plaster casts/immobility* • Shortness of breath of abrupt onset (occasionally gradual onset occurs with multiple small emboli) •Cough *Major risk factors CUM5 2/15/05 4:36 PM Page 70

70 Paper 1 EMQs

• Postoperative/trauma*(hypercoaguable state, An assessment of basic investigations dehydration) •White cell count; to help exclude infection from the • Previous deep vein thrombosis (DVT)/PE* differential diagnosis • Pregnancy* •Arterial blood gases; type I respiratory failure pattern

• Poorly (major illness)* (hypoxia with normal or reduced CO2) ANSWERS 1 ANSWERS •Oral contraception/oestrogens • ECG •Obesity •Tachycardia is the most frequent finding •Smoking •Right axis deviation •Carcinoma •The classic S1Q3T3 is far less common •Travel •D-dimer •Old age (PE is rare under the age of 40 years without •Sensitive but not specific risk factors) •Good for excluding PE if normal •Hereditary thrombophilic states: • Should not be performed if there is strong clinical •Protein S deficiency suspicion •Protein C deficiency •Chest radiograph •Antithrombin III deficiency • Often normal •Factor V Leiden (fails to bind Protein C) •Wedge-shaped infarction (opacity) which may cavi- •Secondary to other medical conditions: tate after several days •Nephrotic syndrome (renal loss of anti-thrombin III) Assessing the probability of PE •Diabetic ketoacidosis (dehydration) This is assessed by scoring the above as follows... •Antiphospholipid syndrome (poorly understood •No other likely diagnosis on the basis of the above mechanism;antibodies to phospholipids involved in clinical findings and the basic investigations? coagulation occur, the APTT is prolonged in vitro Æ 1 point but in vivo thrombosis occurs) •Presence of a major risk factor (as above in bold)? •Homocystinuria Æ 1 point •Paroxysmal nocturnal haemoglobinuria •Behçet’s disease (arterial and venous thromboses) Definitive tests • V/Q scan An assessment of the clinical findings •Helical CT scanning •Tachycardia (sensitive sign) •Pulmonary angiography (the most accurate test) •Tachyopnoea (sensitive sign) Following the diagnosis of PE anticoagulation is com- •Cyanosis menced with warfarin by giving a loading dose of •Chest findings (often normal): 5–10 mg for 2 days,followed by a dose adjusted to give an •Pleural rub INR between 2¥ and 3¥ normal. Warfarin is continued •Reduced air entry •Hypotension •Clinical evidence of DVT (frequently lacking) *Major risk factors

2 points 1 Point 0 points Action (high risk) (medium risk) (low risk)

Heparinize? Yes Yes Wait Arrange definitive tests? Urgent Soon Possibly Alternative diagnosis? Possible Likely Very likely CUM5 2/15/05 4:36 PM Page 71

Paper 1 EMQs 71

for 3–6months.If the PE is a recurrent episode,consider- pleuritic on closer questioning. Diaphragmatic pleural ANSWERS 1 ation is given to lifelong warfarin treatment. Following pain may radiate to the epigastrium and cause con- massive PE with collapse, intravenous thrombolysis is fusion with an acute abdomen. Pneumonia is an given using rtPA or streptokinase. inflammation of the lung. The causes may be divided into . . . (2) None of these •Infective Tension pneumothorax on the left pushing all the medi- •Bacterial,viral,fungal astinal structures over to the right. Patients with tension •Allergic pneumothorax have respiratory compromise, frequent- •e.g.allergic bronchopulmonary aspergillosis ly extreme and rapidly progressive as the ‘valve’type leak •Physical agents causes further compromise with each breath. The im- •Chemical; aspiration of vomit or caustic material pedance of venous return to the right side of the heart •Radiotherapy leads to reduced cardiac output (tachycardia and hy- Remember the right lower lobe as the most likely loca- potension) and an elevated JVP. The management in- tion of a pneumonia following aspiration. volves urgent release using a green needle followed by insertion of an intercostal drain. (See also OSCE 12, (6) Asthma p. 244) Beware similar but late acute asthma with respiratory failure and silent chest.Know the blood gases patterns in (3) Left apical lung fibrosis acute asthma;

• Type I respiratory failure early with low PaO2 and Causes normal (or even elevated) PaCO2 progressing to fa- •Old TB tigue and eventual;

•Silicosis • Type II respiratory failure and rising PaCO2 and low •Sarcoidosis PaO2.In asthma, acidosis is an indication that urgent •Aspergillosis respiratory support (ventilation on ITU) is needed im- •Extrinsic allergic alveolitis mediately, so call for anaesthetic help. (See EMQ An- •Ankylosing spondylitis swers Q24 (4),p.158 and OSCE 1,p.219.)

Differential on CXR of apical opacity •Pancoast tumour Answer 4 •TB •Pneumonia Notes • Klebsiella • Extrinsic pathway:utilizes factor VII. Tissue factor •Pneumocysti initiates coagulation • Fibrosis (as above) • Intrinsic pathway:utilizes factors VIII,IX and XI.Tis- sue factor/factor VII complex activates (4) Right-sided pleural effusion • Common pathway:Factor X dependent Stony dull percussion. Normal respiratory rate and •The PT (prothrombin time) measures factors II, lack of cyanosis makes pneumonia less likely. Know V, VII, and X, i.e. the extrinsic and common causes of effusions (exudates and transudates), CXR ap- pathways pearances of and basic management. (See OSCE 26, • APTT (activated partial thromboplastin time) meas- p. 271.) ures VIII, X and XI, i.e. the intrinsic and common pathways (5) Right lower lobe pneumonia See diagram overleaf. The patient is breathless, cyanosed with evidence of right lower lobe consolidation. Other clinical features might include pyrexia, tachycardia, herpes labialis. The patient might complain of chest pain that is CUM5 2/15/05 4:36 PM Page 72

72 Paper 1 MCQs

PT APTT

EXTRINSIC SYSTEM INTRINSIC SYSTEM

ANSWERS 1 ANSWERS Tissue thromboplastin Factors XII, XI, IX, VI (expressed on surface of) (activated by blood coming into perivascular tissue cells contact with a non-endothelial + surface) + Factor VII Platelet phospholipid + Calcium + Factor VIII COMMON PATHWAY

Factor X Factor V Calcium

Prothrombin Thrombin

Fibrinogen Fibrin

• Vitamin K is a co-factor required for the gamma car- •Bleeding into mucus membranes boxylation of factors II, VII, IX and X, and proteins S •Heavy periods and C. This is required so that these factors can bind •Prolongation of the bleeding time calcium in the normal process of coagulation. The in- Note that the platelet count must drop below around trinsic and extrinsic system will thus be affected by 50 ¥109 before any bleeding disorder is likely to occur. deficiency of vitamin K. Warfarin inhibits the reduc- tion of Vitamin K epoxide to Vitamin K. As Vitamin K Therefore: is a fat soluble vitamin (along with vitamins A, D and (1) Haemophilia A E) deficiency may arise with cholestatic jaundice.Vita- Prolongation of the APTT with normal PT and bleeding min K deficiency can also affect the newborn (haemor- time. Although the inheritance is X linked recessive, one rhagic disease of the newborn) due to poor placental third of haemophilia A cases are spontaneous mutations transfer of the vitamin. For this reason vitamin K is without family history.There is a range of severity.Factor given immediately after birth VIII:C levels less than 5% are associated with sponta- • Coagulation disorders produce: neous haemarthrosis and bleeding into muscles. Restor- •Bleeding into muscles and joints ing factor VIII:C levels to 30% of normal is required in • Large bleeds into the skin the management. •Small cuts stop spontaneously but larger deeper cuts do not (2) Haemophilia B, factor IX deficiency •Healing is delayed (Christmas disease) • Platelet disorders produce: As for Haemophilia A,there is prolongation of the APTT, •Multiple small areas of bleeding into the skin (pur- with normal PT and bleeding time. Factor VIII:C levels pura/petechiae) are normal, however. There is identical inheritance and •Prolonged bleeding of smaller cuts (which depend presentation to Haemophilia A,but the condition is 20% on platelet adhesion more than coagulation) as common as Haemophilia A. CUM5 2/15/05 4:36 PM Page 73

Paper 1 EMQs 73

(3) Von Willebrand’s disease most certainly be investigated for both these possibilities. ANSWERS 1 Deficiency in factor VIII:WF. Three types occur (Types I Note that pregnancy does not contraindicate emergency and II are dominant; Type II is recessive and produces a radiography. Other investigations include urgent blood severe form of the disease). Chromosome 12. The com- cultures, FBC, U and Es, and arterial blood gases. Intra- monest coagulation disorder.VIII:WF binds platelets,so venous fluids and road spectrum antibiotics (e.g. ce- deficiency behaves like platelet problem. Confusingly furoxime 1.5 g t.d.s., plus erythromycin 50 mg q.d.s.) are factor VIII:C (haemophilia A) may (or may not) also be required.Inatropic support may be required on ICU. low,with a resultant prolongation of the APTT.However, haemarthrosis is rare.Ristocetin (an antibiotic) normal- (3) Epidemic (Bornholm’s disease) ly clumps platelets, but fails to do so in Von Willebrand’s Coxsackie B virus. A typical history. A seasonal illness disease and so forms the basis of a the diagnostic test. often occurring in late summer. The pain may be unilat- eral and severe (‘the devil’s grip’). There are no signifi- (4) Warfarin medication cant clinical findings on examination. Symptomatic Although Vitamin K deficiency will produce the same treatment is all that is required and resolution occurs pattern. The raised PT and APTT in this case are sugges- spontaneously. tive of warfarin therapy given the clinical scenario. The normal bleeding time counts against liver disease, (4) Aortic dissection of the thoracic aorta which causes a coagulopathy via... The patient has a history of hypertension, which is a •Impaired protein synthesis; fibrinogen/coagulation major risk factor. Often a history of a valsalva type ma- factors noeuvre (e.g. lifting, bending) before the event is ob- •Malabsorbtion of fat soluble vitamin K due to tained. The pain is usually very severe and described as cholestasis ‘ripping’ or ‘tearing’ classically. Two thirds are of the as- •Hypersplenism; consumes platelets cending aorta, one third are in the descending aorta. • DIC Clinical examination is frequently normal, the diagnosis •Functional platelet abnormalities. Also seen in being made from the history and absence of the acute uraemia/myeloma/scury/antiplatelet drugs/essential ECG changes of myocardial infarction (the major differ- thrombocythemia/leukaemia/congenital conditions ential diagnosis).

(5) Easy bruising syndrome Risk factors for aortic dissection Common and benign. Reassurance required. Similar to •Hypertension senile purpura in that vessel fragility is to blame. All •Coarctation of the aorta (Turner’s Syndrome; XO) measurements of coagulation are normal. •Marfan’s syndrome • Giant cell arteritis Answer 5 •Major thoracic trauma

(1) Tietze’s syndrome Complications for dissection of the aorta Costochondritis of the sterno-costal junctions. The •Inferior MI (occlusion of the right coronary artery ori- aetiology is uncertain. Symptomatic treatment and gin) reassurance is all that is required. •Aortic regurgitation (causing cardiac failure) • (retrograde dissection) (2) Lobar pneumonia •Dissection of the major arteries coming off the aortic The main differential being pulmonary embolism, given arch leading to strokes and limb ischaemia. the patient’s clear major risk factors for thrombosis. •Rupture of the false lumen back into the aorta or There has been no haemoptysis (although this does not externally (usually resulting in death). In this case exclude a PE) and septic features dominate the clinical pulmonary haemorrhage had occurred picture. In addition the prodromal symptoms and the ‘gradual’ onset of breathlessness make pneumonia Investigations the more likely diagnosis. She is clearly very unwell given • ECG: non-specific changes e.g. tachycardia, left ven- the hypotension (septic shock) and in practice would al- tricular strain,possible inferior MI CUM10 2/15/05 4:34 PM Page 229

OSCEs 229

OSCE station 3 Q. What are the causes of an enlarged kidney? A. Q. What are the causes of a mass in the right iliac fossa? •Renal carcinoma A. •Hydronephrosis •Carcinoma of the caecum •Hypertrophy of a solitary kidney •Crohn’s disease (usually an underweight, frequently •Adult polycystic kidney disease young patient with scars from previous resections) •May have had a previous nephrectomy (for a painful •Appendix mass or abscess non-functioning kidney) so that only one kidney is •Lymphoma palpable •Ileocaecal TB •Cysts develop from all segments of the nephron dur- •Ovarian tumour (NB ovarian tumours often rise out of ing adolescence. Patients present with renal pain, the pelvis centrally) treatment resistant hypertension, haematuria and •Transplanted kidney stone symptoms •(Amoebiasis) •Renal failure occurs later in life (10% of chronic renal • (Schistosomiasis) failure) •(Carcinoid tumour) •Various associations exist [see EMQ Answers 19 (2), p. 142] Q. What would be your next investigation for this right iliac fossa mass? Q. What are the symptoms and signs of uraemia? A. A. •Plain abdominal radiograph,looking for . . . Clinical features of uraemia . . . •Obstruction (air/ fluid levels) •Symptoms •Calcification (tuberculosis) •Nausea,vomiting •Bowel wall oedema thickening (CD) •Diarrhoea •Chest radiograph •Fatigue • TB •Breathlessness: •Metastatic disease Anaemia •Abdominal ultrasound scan Pleural effusion •Abdominal CT scan Pericardial effusion •Small bowel meal and follow through for Crohn’s Impaired ventricular function disease •Weakness •Diagnostic laparoscopy •Itching •Signs Q. What are the causes of a mass in the left iliac fossa? •Pallor (anaemia) A. • Scratch marks (pruritus) •Colonic carcinoma •Bruising •Diverticular mass •Pericarditis (pericardia rub or effusion) •Transplanted kidney •Hypertension •Ovarian tumour •Oedema •Faeces •Confusion •Muscle weakness Q. What are the causes of an epigastric mass? •Fistula (for dialysis) A. OSCEs •Hepatomegaly (see OSCE 4,p.230) • Gastric malignancy •Pancreatic •Pseudocyst •Malignancy •Lymphoma CUM10 2/15/05 4:34 PM Page 230

230 OSCEs

OSCE station 4 Hepatitis A,B (D),C Leptospirosis (Weil’s disease) Q. What are the causes of hepatomegaly (without Infectious mononucleosis splenomegaly)? Others: A. TB,syphilis,brucellosis • Malignancy Tropical infections,e.g.malaria,kala-azar • Secondary tumours mainly •Haematological malignancy: •Primary hepatocellular carcinoma CML* and CLL* •Lymphoma Polycythemia vera •Leukaemias: Myelofibrosis* CML Lymphoma CLL •Haemolytic anaemias (splenic filtration) Myelofibrosis • Others • Cirrhosis •Connective tissue disease: • NB Late alcoholic cirrhosis causes a small fibrotic SLE liver RA • Blood •Sarcoidosis • Cardiac failure •Amyloidosis •Tricuspid regurgitation (pulsatile liver) •Budd Chiari syndrome (hepatic vein thrombosis) Q. What are the causes of hepatosplenomegaly? • NB Hepatic engorgement can give abnormal LFTs A. (obstructive picture) As above; however, haematological malignancy and in- • Bile fections feature more prominently. •Primary biliary cirrhosis •Common bile duct obstruction: Q. How can you differentiate an enlarged spleen from an Stones enlarged liver? Strictures A. Tumours There is a mass in the left/ right hypochondrium extend- Lymph nodes ing down towards the pelvis.It moves with respiration.It • Infections is resonant to percussion (over lying air filled bowel).The • Viral hepatitisA,B (D),C upper border can be palpated (unlike a spleen or •Leptospirosis liver which have their upper pole under the ribs).You can •Infectious mononucleosis ballot the mass bimanually assessing the size, shape and •Others: consistency.Look for abdominal scars (?nephrectomy on TB,syphilis,brucellosis the opposite side). Look also for an AV dialysis fistula Tropical,e.g.malaria,kala-azar over the distil radius. • Infiltrations •Amyloid •Sarcoid Kidney Spleen •Wilson’s disease •Haemachromatosis Enlarges? Inferiorly To RIF •Glycogen storage diseases Ballotable? Yes No Resonant? Yes No Q. What are the causes of splenomegaly? Get above? Yes No A.

OSCEs • Blood •Portal hypertension (late cirrhosis with a small liver) • Busy spleen *Cause massive splenomegaly (as likely to be found in an •Infections: examination) CUM10 2/15/05 4:34 PM Page 231

OSCEs 231

OSCE station 5 Have you noticed any changes in your vision? ‘I do occasionally suffer a bit of double vision, mainly A 48-year-old female sits in front of you in your OSCE. when I’m tired.’ You are asked to take a thyroid-based history from her. Have you noticed any changes in your periods? ‘Yes, they’re becoming much more infrequent. Very Q. What questions would you ask her? hit-and-miss. But I suppose I’m at that time of life, A. aren’t I?’ Has your weight changed recently? ‘Oh yes, I’ve lost over a stone in the last 3 months You are asked to perform a thorough examination of the alone.’ person’s thyroid status. Have you been trying to lose weight? ‘No.’ From the end of the bed you note a thin-looking woman Has your appetite changed? who is very jittery and fidgetsincessantly with her hands. ‘It has.I eat like a horse and yet I continue to lose all this You ask her to hold her hands out straight in front of her weight. In fact, I’m getting a bit concerned, as this is and close her eyes.You place a sheet of paper on top of her what happened to my mother shortly before she was hands to accentuate the very fine tremor that is present. diagnosed with cancer.’ Her hands are warm and her palms are flushed.There is Have you noticed any change in your behaviour? neither clubbing, swollen fingers nor periosteal bone ‘Not really, but my husband says that I’ve changed of formation. You palpate her pulse, and note a rate of late; that I’m crabby and irritable all the time. I just 98/min, irregularly irregular.Blood pressure is 161/91. thought it was “the change”, you know. He also says JVP is not raised. In her face you note exophthalmos, that I’m unable to sit still and I suppose he’s right and to confirm this you stand behind the patient, look- about that really.I just like to keep active.’ ing down from on top of their forehead to check that Have you noticed any weakness of your muscles? there is genuine protrusion if the eyeballs. You check ‘Not as such,though in saying that,I have noticed that for lid lag and there is none present. There is no clear of late I’ve found it a bit of a trial climbing up the stairs, loss of acuity, ophthalmoplegia or conjunctival oedema, especially if I’m carrying the washing.’ and no goitre in the neck. The apex beat is normally Have you noticed any trembling of your fingers or located, but it is hyperdynamic. are nor- hands? mal and the chest is clear.There is no peripheral oedema, ‘It’s strange you should say that, because just the although there is evidence of a rash on both shins. Com- other day I was writing a letter and I noticed that I prehensive neurological examination reveals question- couldn’t stop my hand trembling. Now I’m beginning able wasting of the limb girdle musculature, and to notice it more and more, even when I’m holding a decreased power in both pectoral and pelvic muscles cup of tea.’ (MRC grade 4). Have you had any palpitations? ‘All the time. But that’s nothing new, I’ve had them for Q. What is the cause of this woman’s clinical situation? years.I’ve got an irregular heart beat,you know.’ A. Are you as a rule a ‘hot’or a ‘cold’person? This woman is hyperthyroid and is displaying nearly all ‘I’m always red hot. And sweat? I could sweat for Eng- of the characteristic signs and symptoms.Like all types of land, even when everyone else in the room is perfectly thyroid disease,hyperthyroidism is more common in fe- happy with the temperature.’ males,patients with autoimmune disease and those with Have you noticed any change in your bowel habit over a family history of thyroid disease.

the last year or so? OSCEs ‘Well I am a bit “looser” than I used to be in that Q. What are the common causes of hyperthyroidism? department.’ A. Have you noticed any change in your appearance? The commonest cause of hyperthyroidism, and the only ‘Well my son says that my eyes are “goggly”,but I don’t one associated with thyroid eye disease, is Graves’ dis- really see any change.’ ease.It is due to autoantibodies directed at the TSH re- CUM10 2/15/05 4:34 PM Page 232

232 OSCEs

ceptor.It typically causes a fluctuating clinical picture, Q. What investigations would you perform to confirm with alternating relapse and remission. your diagnosis? A. Other causes of hyperthyroidism include . . . In all cases of primary hyperthyroidism, TSH will be • Toxic multinodular goitre:difficult to treat,tradition- suppressed,and this is the initial investigation of choice.

al antithyroid drugs rarely successful When this is combined with a raised serum T3 or T4, • Single toxic adenoma (Plummer’s disease): similarly then the diagnosis of hyperthyroidism is made. TSH- difficult to treat receptor autoantibodies will be present in cases of • Iatrogenic hyperthyroidism (excessive levothyroxine Grave’s disease, but these are not commonly measured. replacement) Surprisingly,the microsomal and thyroglobulin autoan- • Drug-induced hyperthyroidism (e.g. due to lithium, tibodies associated with hypothyroidism are also often amiodarone) found in Grave’s disease. • Transient thyroiditis (e.g. de Quervain’s viral-medi- In very rare situations, a pituitary tumour will be the ated transient thyroiditis,post-partum thyroiditis) cause of hyperthyroidsim, and TSH will be raised. The • Thyrotoxicosis factitia (patient takes thyroxine, but differing laboratory findings in thyroid diseases are sum- attempts to conceal this) marized in the table below...

Thyroid state Thyroxine (T4)Triiodothyronine (T3)TSH (pituitary) TRH (hypothalamus)

Hyperthyroidism Graves’disease Raised Raised Decreased Decreased Toxic multinodular goitre Raised Raised Decreased Decreased Pituitary tumour Raised Raised Raised Decreased

Hypothyroidism Primary Decreased Decreased Increased Increased Secondary Decreased Decreased Decreased Increased Tertiary Decreased Decreased Decreased Decreased

Q. What are the treatment options available? neoplastic causes of hyperthyroidism because of the A. morbidity, expense and free availability of alternative ef- Treatment of hyperthyroidism is complex, and depends fective treatments.The procedure of choice is a sub-total on the underlying cause and the clinician’s choice. In thyroidectomy,and this should only be performed in pa- general,however,it can be split into three options... tients rendered euthyroid with antithyroid drugs. The 1. Radioiodine:a safe treatment option that usually drugs should be stopped 2 weeks before surgery and leads to euthyroidism in 3–6 months. I-131 is given and it potassium iodide given to reduce the vascularity of the accumulates in the thyroid,destroying the gland.The pa- gland tient should be rendered euthyroid before treatment is given,with anti-thyroid medicaitons Whatever treatment option is chosen b-blockers are a 2. Antithyroid drugs,e.g. carbimazole, propylth- useful adjunct to treatment because they block the sym- iouracil, methimazole: if these are going to work, they pathetic side effects that are so distressing in hyperthy- will usually restore a euthyroid state in 4–12 weeks.They roidism. Propranolol also has the ability to block

OSCEs are less likely to lead to permanent remission than either peripheral conversion of T4 to T3. of the other main treatment options, and long-term therapy may be necessary. Side effects can be severe, and Q.What is thyroid storm? include hepatotoxicity,vasculitis and agranulocytosis A. 3. Surgery: the least favoured treatment option of non- This is also known as thyrotoxic crisis, and is a condition CUM10 2/15/05 4:34 PM Page 233

OSCEs 233

with a 20–50% mortality,despite optimal management. to push in on her hips to tense the pectoralis It occurs as an exaggerated manifestation of hyperthy- musculature roidism, and is precipitated by major stressors,such as •With the patient leaning forward slightly infection, trauma, major surgery, diabetic ketoacidosis, • On inspection you are looking for a number of MI, CVA, PE and withdrawal of thyroid medications. It things... presents with an overactivated sympathetic nervous •Obvious lumps or masses system (palpitations,tachycardia,tachyarrhythmia,car- • Scars (previous neoplastic, augmentative or reduc- diac failure, anxiety, agitation), and typically a very high tive surgery) fever.Jaundice is a late and ominous sign. •Sinuses •Bruising (old fine needle aspiration site) Diagnosis should be made clinically, as thyroid function •Inflammatory changes in skin (peau d’orange) tests often fail to be grossly deranged.Treatment is... •Colour of skin (if darkened, may suggest previous • ALS stabilization of airway,breathing and circulation radiotherapy) •Commencement of an intravenous fluid infusion •Irregular skin thickening • b-blocker infusion to block adrenergic effects •Retraction • Cooling blankets •Dimpling • Paracetamol •Asymmetry

• Propylthiouracil to block production of new T4 •Nipple changes [e.g. indrawing, eczema (Paget’s dis- • Iodine to block release of pre-formed T4 ease),differing directions,inversion] •Lactation Do not give the iodine until the propylthiouracil has •Fungation or ulceration had chance to work (90 min) otherwise there is a risk of •Move on to palpation of the breast tissue itself. The making the thyroid storm worse. Treat any precipitating best position to have the patient in is supine, with the factors that may be present. hand behind the head. •There are a number of ways to palpate the mass of the breast; which one you use does not matter as long OSCE station 6 as you cover the whole of the breast tissue.Expect the palpation of a single breast to take 3minutes. You arrive at an OSCE station and are asked to examine a Either . . . patient’s breasts(this will almost always be on a dummy; • Split the breast into four quadrants and palpate each hence, you would be expected to both show what you quadrant in turn using small circular movements would do and describe what you would do because the with the pads of the fingers, about the size of a 50 limitations of the model mean that you cannot physically pence piece; or ... demonstrate what you would like the patient to do). • Start just anterior to the axilla and circle around the • Introduceyourself to the patient,stating full name and periphery of the breast in an ever-decreasing circle; grade or ... • Explainwhat it is that you propose to do,and that there • Start just anterior to the axilla and palpate circularly will be a nurse chaperone present at all times up and down the breast tissue moving from lateral to • Gain consent medial •Give the patient time to get undressed in private if she is •Ifyou find any abnormality you must describe its... not already de-robed • Site on the breast (and on which breast):often giving • Ensure that you have a female nurse chaperone a time in the ‘o’clock’ form helps; also site within the • Once undressed, uncover the patient from lower ribs breast (superficial or deep) upward • Size in cm OSCEs •Begin with inspection:inspect in a number of ways • Shape (circular,ovoid,elongated) ... • Surface (regular or irregular) •With the patient sitting comfortably at rest with her • Consistency (soft or hard) by her sides • Tender or non-tender •With the patient’s hands in the air above her head •Overlying skin (tethered or mobile) •With the patient’s hands on her hips; ask the patient •Underlying fascia (tethered or mobile) CUM10 2/15/05 4:34 PM Page 234

234 OSCEs

• Temperature (any evidence of inflammation) starting as high in the axilla as you can, palpate the • Transillumability (can you shine a light through it medial and lateral axillary walls, attempting to trap or not) any nodes against the chest wall or medial humerus • Once you have completely described it,move on to pal- respectively as you do so. Warn the patient that this pate the rest of the breast tissue from where you left off; part of the examination may be the least comfortable just because you have found one abnormality does not •Describe any abnormalities found in the same way as mean there are not more to find (there usually are in for a breast lump the OSCE)! •Complete your palpation by feeling for any enlarged •Describe each abnormality in the same way supraclavicular lymph nodes from behind • Once you reach the nipple,you must bi-manually pal- • Immediately cover the patient up, thank them and pate it by placing two fingers either side and gently turn to the examiner to present your findings rocking the nipple,with the specific purpose of palpat- Be aware that you may get asked to examine a patient who ing any masses behind the nipple, and also of express- has had a previous mastectomy.Do not panic! Begin by ing any abnormal secretions or galactorrhoea from the inspecting the mastectomy scar and axilla, looking for nipple itself signs of masses or nodularity. Note especially the colour •Move on the axilla (it is best to sit the patient up again of the skin (radiation) and the presence of lymphoedema to do so)... (impaired lymph drainage post-surgery). Move on to •Begin with inspection,looking for signs of masses, palpation:palpate gently along the scar (it is often exquis- infection (hidradenitis suppuritiva), rashes or un- itely tender), and then move on to the rest of the chest usual skin pigmentation (acanthosis nigricans) wall in a similar way as if the breast was still present. •Palpate any breast tissue you suspect to be deep in the Take special care in palpating the axilla, looking for axilla (do not forget that the axillary tail of Spence lymphadenopathy. can reach far into the axilla in some women) •Then palpate for lymph nodes.Support the arm of The way to differentiate between the most common three the patient with your non-examining hand, and, breast masses is shown in the table below...

Fibroadenoma Breast cyst (breast mouse) Carcinoma

Age of patient Early middle age,aged Young,aged 15–30; rare after Older,aged 35–100+;usually 35–50 years; uncommon age 50 years patient is >50 years post-menopause except in women on HRT

Shape of lesion Spherical Spherical,discoid or lobular Irregular

Consistency of lesion Soft Firm Hard

Surface of lesion Delineated from Delineated from surrounds NOT delineated from surrounds surrounds

Tender or not Usually Rarely Rarely

Tethering Moderately mobile Very mobile Commonly tethered to skin or fascia OSCEs Number of lesions Commonly multiple Can be multiple,but usually Usually single at one time single Continued